What Is the Impact of Mapping in Linear Transformations from P2 to P3?

Click For Summary
The discussion focuses on the linear transformation T: P2 --> P3, which maps a polynomial p(t) to (t+5)p(t). The image of the polynomial p(t) = 2 - t + t^2 is calculated as 10 - 3t + 4t^2 + t^3. There is confusion regarding the outputs T(1) and T(t), with participants clarifying that T(1) = t + 5 and T(t) = t^2 + 5t. The transformation's matrix representation relative to the specified bases is also discussed, emphasizing the importance of correctly interpreting the transformation results. Understanding the mapping and matrix representation is crucial for grasping the implications of linear transformations in polynomial spaces.
Pouyan
Messages
103
Reaction score
8

Homework Statement



Let T: P2 --> P3 be the transformation that maps a polynomial p(t) into the polynomial (t+5)p(t).

a) find the image of p(t)= 2-t+(t^2)
b) Find the matrix for T relative to bases {1,t,t^2} and {1,t,t^2,t^3}.

Homework Equations


Given

The Attempt at a Solution


a) I know (t+5)p(t)=(t+5)(2-t+(t^2))= 10-3t+4(t^2)+(t^3)

b) I see in solution T(1) = (t+5) (1)= t+5
T(t) = (t+5)(t)
T(t^2)=(t+5)(t^2)
and so on ...

My question is why T(1)= 1 and T(t) = t ?! I see that T(1) means p(t)=1 or T(t)=p(t)=t but why is this so ?!
 
Physics news on Phys.org
Pouyan said:

Homework Statement



Let T: P2 --> P3 be the transformation that maps a polynomial p(t) into the polynomial (t+5)p(t).

a) find the image of p(t)= 2-t+(t^2)
b) Find the matrix for T relative to bases {1,t,t^2} and {1,t,t^2,t^3}.

Homework Equations


Given

The Attempt at a Solution


a) I know (t+5)p(t)=(t+5)(2-t+(t^2))= 10-3t+4(t^2)+(t^3)

b) I see in solution T(1) = (t+5) (1)= t+5
T(t) = (t+5)(t)
T(t^2)=(t+5)(t^2)
and so on ...

My question is why T(1)= 1 and T(t) = t ?! I see that T(1) means p(t)=1 or T(t)=p(t)=t but why is this so ?!
T(1) isn't equal to 1, nor is T(t) equal to t. Why do you think they are?
 
  • Like
Likes Pouyan
vela said:
T(1) isn't equal to 1, nor is T(t) equal to t. Why do you think they are?
I see in my solution B={1,t,t^2} and C={1,t,t^2,t^3} Since T(b1)=T(1)=(t+5)(1)=t+5, [T(b1)] relative to C =[5,1,0,0,]
 
Question: A clock's minute hand has length 4 and its hour hand has length 3. What is the distance between the tips at the moment when it is increasing most rapidly?(Putnam Exam Question) Answer: Making assumption that both the hands moves at constant angular velocities, the answer is ## \sqrt{7} .## But don't you think this assumption is somewhat doubtful and wrong?

Similar threads

  • · Replies 24 ·
Replies
24
Views
3K
Replies
5
Views
2K
  • · Replies 7 ·
Replies
7
Views
1K
  • · Replies 4 ·
Replies
4
Views
3K
  • · Replies 1 ·
Replies
1
Views
1K
  • · Replies 3 ·
Replies
3
Views
2K
Replies
2
Views
2K
Replies
4
Views
2K
  • · Replies 1 ·
Replies
1
Views
1K
  • · Replies 1 ·
Replies
1
Views
2K